Math pls help for 26 points

Math Pls Help For 26 Points

Answers

Answer 1

Answer:

77

Step-by-step explanation:


Related Questions

Solve the quadratic equation 3x^2+8x=30 by completing the square.

Which expression correctly represents the solutions of the equation?

4±√74/3
-4±√106/3
8±2√106/6
-3±√26/3

Answers

I think the second one

Answer:

B

Step-by-step explanation:

im bad at explaining this :(

do all of the digits in 8.88 have the same value? and explain why.

Answers

Answer:

No

Step-by-step explanation:

Mitochondria is the powerhouse of the cell

A right triangle has leg lengths of 3.4 cm and 6.7 cm. Which answer below shows the length of the triangle’s hypotenuse rounded to the nearest tenth? A. 8.5 cm B. 7.5 cm C. 5.5 cm D. none of the above

Answers

Answer:

The answer is B

Step-by-step explanation:

What is the congruence statement?

Answers

Triangle JPA is congruent to triangle TVS

Answer:

triangle pjl is congruent to it's by side side side theorem

Find the slope of the line that passes through (9, 8) and (6, 13).
Simplify your answer and write it as a proper fraction, improper fraction, or integer.

Answers

Answer:

-5/3

Step-by-step explanation:

Answer:

slope: -1.6666666

Step-by-step explanation:

Use the formula m=(y2-y1)/(x2-x1)

Substitute values: m=(13-8)/(6-9)

Solve that, and you will find m (slope) = -1.666666

Express the numbers in scientific notation:
1. 75000
2. 0.0024
3. 75,000,000
4. 0.0000024

Answers

Answer:

1. 75000 7.5 × [tex]10^{4}[/tex]

2. 0.0024 ⇒ 2.4 × [tex]10^{-3}[/tex]

3. 75,000,000 ⇒ 7.5 × [tex]10^{7}[/tex]

4. 0.0000024 ⇒ 2.4 × [tex]10^{-6}[/tex]

Step-by-step explanation:

Scientific notation is a way of writing very large or very small numbers.

A number is written in scientific as a number between 1 and 10 and multiplied by a power of 10

The form of the scientific notation is a × [tex]10^{n}[/tex], where

1 < a < 10n is an integern is positive if the decimal point moves from right to leftn is negative if the decimal point moves from left to right

1. 75000

→ Place a decimal point between 7 and 5 to make the number between

   1 and 10

a = 7.5

→ Count the number of the digits right to the decimal point

∵ There are 4 digits right to the decimal point

n = 4

75000 ⇒ 7.5 × [tex]10^{4}[/tex]

2. 0.0024

→ Move the decimal point from left to right to place it between 2 and 4 to

   make the number between 1 and 10

a = 2.4

→ Count the number of the digits left to the decimal point

∵ There are 3 digits left to the decimal point

n = -3

0.0024 ⇒ 2.4 × [tex]10^{-3}[/tex]

3. 75,000,000

→ Place a decimal point between 7 and 5 to make the number between

   1 and 10

a = 7.5

→ Count the number of the digits right to the decimal point

∵ There are 7 digits right to the decimal point

n = 7

75,000,000 ⇒ 7.5 × [tex]10^{7}[/tex]

4. 0.0000024

→ Move the decimal point from left to right to place it between 2 and 4 to

   make the number between 1 and 10

a = 2.4

→ Count the number of the digits left to the decimal point

∵ There are 6 digits left to the decimal point

n = -6

0.0000024 ⇒ 2.4 × [tex]10^{-6}[/tex]

Solve for x in the figure below.

Answers

Answer:

20°

Step-by-step explanation:

180° - 112° = 68°

68° + 3x + 4° + 6x = 180°

72° + 9x = 180°

9x = 108°

x = 20°

Round your answer to the nearest tenth of a degree.

Answers

Answer:

136.5

multiply 21 X 13 then divide by 2

Wilscy owns a toy store. The number of toy trains sold (in thousands) from 2000 through 2005 can be modeled by M(x) = −0.5x3 + 2x2 − x + 9. The average cost per toy train (in dollars) can be modeled by N(x) = 0.3x + 7, where x represents the number of years since 2000. The total revenue Wilscy made by the sale of toy trains is the product of the number of toy trains sold and the average cost per toy train. Identify a polynomial P(x) that can be used to model the total revenue.

Options:
a: P(x) = −0.15x4 − 2.9x3 + 13.7x2 − 4.3x + 63
b: P(x) = −0.5x3 + 2x2 − 0.7x + 2
c: P(x) = −0.15x4 + 4.1x3 + 14.3x2 − 9.7x + 63
d: P(x) = −0.5x3 + 2x2 − 1.3x + 16

Answers

Answer: Hey there! The answer is A!

Step-by-step explanation:

Base salary is 100 percent, added bonus is 6 percent, total salary is 106 percent.
Use the diagram to help you solve the problem.

Mark is a receptionist for Exclusive Auto Sales. His base salary is $21,000. He has a good year of customer service, so the company also gives him a 6 percent year-end bonus based on his salary. How much does Mark make, including his base salary and the year-end bonus?

Answers

Answer:

22,260

Step-by-step explanation:

Answer:

i just took the answer thing and it would be 22,260 sorry i am so late with the answer but yea hope this helps

In the diagram, Find the value of y

Answers

Answer:

y = 42

Step-by-step explanation:

we can start by getting teh degree of x

62 + 65 = 127

180 - 127 = 53

x = 53

x - y = 11

we know that 53 is x

53 - y = 11

y = 42

Marie gets paid $ 2,000 monthly and a commission of 50 dollars for each sale she makes. Write an expression that represents her salary.

Answers

Answer:

2000+(50× x)

Step-by-step explanation:

The certain salary is 2000 but we do not know how much sale where made so the amount is placed as x inwhich will be multiples by 50 then added to 2000 to get total

What form is the equation below written in?
y= (x - 2)2 – 9

-Factored Form
-Standard Form
-Slope-Intercept Form
-Vertex Form

Answers

Answer:

            Vertex Form

Step-by-step explanation:

Vertex form of quadratic function with vertex (h, k) is y=a(x-h)²+k,

So  y= (x - 2)² - 9 is vertex form with vertex (2, -9)

Find cos(B) in the triangle

Answers

Answer:

what is the question

Step-by-step explanation:

Lines k and I are parallel and cut by transversal j.


Which statement is a valid conclusion?

A.<6 and <7 are complementary

B. <4 and <8 form a linear pair

C. <1 and <2 are congruent

D.
<2 and <7 form vertical angles

Answers

Answer:

D.

<2 and <7 form vertical angles

standard covert 3x-24 = 1 ​

Answers

Answer:

the answer is -9 it think

Find the 52nd term of the arithmetic sequence 5, 9, 13

Answers

Answer:

209

Step-by-step explanation:

You would first create an explicit formula for the provided sequence.

The basic explicit formula for arithmetic sequences is [tex]a_{n} = d(n-1) + a_{1}[/tex], where an is the number of the term, d is the number you are adding or subtracting by, n the location of the term, and a1 is the first number.

We would then substitute the values given into the formula.  

We are trying to solve the value of the 52nd term. This makes n = 52. The first number of the sequence is 5, so a1 is 5. Finally, d is 4 because we are adding 4 to each number in the sequence.

Therefore, our resulting equation would be [tex]a_{n} =4(52-1)+5[/tex], which equals 209.

What is the answer to this question

Answers

Answer:

i think it is....

1

0

-1

-2

-3

sorry if im wrong mark brainliest if righ

can someone please help? i would really appreciate it :)

Answers

Oh this is easy it’s the same angles for the figure to the right to shape is just turned a different way just put the correct degrees in the the right angles

Write 0.15 as a fraction in simplest

Answers

Answer: 3/20 also known as 15%  Hope this helps Have a great night

Step-by-step explanation:

Graph this line using the slope and y-intercept:
y = 8x - 6
Click to select points on the graph.
8
6
4
No
-6
-4
-2
-10
-8
0
co
4
2
6
10
-2
-4
-6
CO
-104

Answers

Here you go. I hope you get it.

The graph of the equation y = 8x - 6 is a straight line.

What is slope intercept form?

The slope intercept formula y = mx + b is used when you know the slope of the line to be examined and the point given is also the y intercept

(0, b).

Given is an equation, y = 8x - 6

Finding the coordinates,

Put x = 0, y = -6

(0, -6)

Put y = 0

8x = 6

x = 0.75

(0.75, 0)

Plotting the graph,

Hence, we will get a straight line of the graph.

Learn more about slope intercept form, click;

https://brainly.com/question/30216543

#SPJ2

GUYS PLEASE HELP ME ive been struggling with this for the past 15 minutes

Answers

Answer:

Loading Answer...

Step-by-step explanation:

Loading Explanation...

Help needed. Only 1-5 is needed help on plz and ty

Answers

Answer:

number 5 is y= 3    7-3=4x4=16

Step-by-step explanation:

what is 1983917481724869826846164619/1816498646149816496416461984628+28319727847124798-274812987198463861294=?​

Answers

Answer:

-2.7478467e+20

Step-by-step explanation:

Ty google :3

I will pick the brainiest answer!!! Thanks! :)

Answers

Answer:

For the first picture:

1) Right angle

2)Acute

3)Obtuse

For the second picture:

W) Acute

Step-by-step explanation:

Question 12 pts
Line a passes through the points (-2, 3) and (4, 6), line b passes through the points (0, 2) and (2, 3). Are these lines parallel?

Group of answer choices

No, they are not parallel

Yes, they are parallel because their slopes are negative reciprocals

Yes, they are parallel because they have the same slope

Not enough information to answer

Flag this Question
Question 22 pts
Write an equation for the line that passes through the point (4, -1) and is parallel to the line y = 2x - 5.

Group of answer choices

LaTeX: y-1=2\left(x-4\right)y − 1 = 2 ( x − 4 )


LaTeX: y+1=2\left(x-4\right)y + 1 = 2 ( x − 4 )


LaTeX: y+1=5\left(x-4\right)y + 1 = 5 ( x − 4 )y + 1 = 5 ( x − 4 )


LaTeX: y-1=5\left(x-4\right)y − 1 = 5 ( x − 4 )y − 1 = 5 ( x − 4 )


Flag this Question
Question 32 pts
Write an equation for the line that passes through the point (-2, -4) and is perpendicular to the line y = 3x - 2.

Group of answer choices

LaTeX: y+4=-\frac{1}{3}\left(x+2\right)y + 4 = − 1 3 ( x + 2 )


LaTeX: y+4=\frac{1}{3}\left(x+2\right)y + 4 = 1 3 ( x + 2 )


LaTeX: y-4=-\frac{1}{3}\left(x-2\right)y − 4 = − 1 3 ( x − 2 )


LaTeX: y-4=\frac{1}{3}\left(x-2\right)y − 4 = 1 3 ( x − 2 )


Flag this Question
Question 42 pts
Write an equation for the line that passes through the point (-1, 5) and is perpendicular to the line LaTeX: y=-\frac{1}{2}x-6y = − 1 2 x − 6

Group of answer choices

LaTeX: y-5=2\left(x-1\right)y − 5 = 2 ( x − 1 )


LaTeX: y-5=-2\left(x-1\right)y − 5 = − 2 ( x − 1 )y − 5 = − 2 ( x − 1 )


LaTeX: y-5=-2\left(x+1\right)y − 5 = − 2 ( x + 1 )y − 5 = − 2 ( x + 1 )


LaTeX: y-5=2\left(x+1\right)y − 5 = 2 ( x + 1 )


Flag this Question
Question 52 pts
Write an equation in standard form for the line that passes through the point (3, -4) and is parallel to the line LaTeX: y=\frac{1}{2}x+8y = 1 2 x + 8

Group of answer choices

LaTeX: x-2y=7x − 2 y = 7


LaTeX: x-2y=-16x − 2 y = − 16x − 2 y = − 16


LaTeX: x-2y=11x − 2 y = 11


LaTeX: x-2y=14

Answers

Answer:

tour answer should be no they are not be parellel

Step-by-step explanation:

Answer:

tour answer should be no they are not be parellel

Step-by-step explanation:

This table shows the input and output values for an exponential function f(x).
x -3 -2 -1 0 1 2 3
f(x) 8 4 2 1 0.5 0.25 0.125
What is the ratio of outputs for any two inputs that are one value apart?


4

0.5

0

1.5

Answers

Answer:

its 0.5 I took the test and it was correct

Step-by-step explanation:

What is the answer to this question

Answers

Answer:

Slope: 6          Y-intercept: 2

Step-by-step explanation:

See where the line hits perfectly at an age and calculate rise/run. Here the rise is 6 and the run is 1, therefore 6/1 = 6.

Where is the y value that equals 0 in the x value? Right in the middle of the y line, so its 2. Hope this makes sense, I’m not too good at explaining stuff lol

which of the following equations would model a population with a initial size of 625 that is growing at an annual rate of 8.5%
1. P=625(8.5)^t
2. P=625(1.085)^t
3. P=1.085^t+625
4. 8.5t^2+625

Answers

Answer:

Believe its B

Step-by-step explanation:

A group of students was asked, "How many hours did you watch television last week?" Here are their responses: 9, 16, 11, 19, 4, 10, 15, 19

Answers

Answer:

So what's your Question.

I think The Mean ????

Step-by-step explanation:

So it is Add all the Values which is 103

Divide it by no. of Values which is 8

You'll get 12

Which is Mean !!!!

Other Questions
Please help due tonight !!!! In which way do REITs resemble mutual funds? Help Im dumb and its the end of the quarter in the movie "gravity" - Why did the escape pod get stuck and pulled back? if Jazzy sells her old bike for $104 and some toys for 68 she uses the money for four month karate lessons the cost of lessons is the same each month which strip diagram shows how to find Jazzy's cost for one month of karate lessons.lend a hand please also this is 4th grade If vector u= (-4,8) and vector (5,-18), which vector can be added to 2u-4v to give the unit vector in the direction of the y-axis as the resultant vector?A. w = (12, -87)B. w = (28, -55)C. w = (28, -87)D. w = (12, -55) Write an equation to match this graph 7 Given: -8(w + 1) = -5(w + 10); Prove: w = 14Statements which rule describes the transformation? picture below PLEASEEEE HELPPP MEEEE! NOWWWWW PLZ HELP the question is: Write an informative essay on the topic of immigration. Your essay will use research to describe the changes and challenges that today's immigrant children experience. You're not allowed to say your experiences or opinion Write at least two feelings using keywords eassy about the impact of social media on tourism industry What did Hannibal do that made him famous? How many parts 3 percent milk and 40 percent cream would be needed to make 500 pounds of 5 percent milk order the numbers from least to greatest 7,-2.5,4/6 I just need help with this please. A construction company can remove 3/4 tons of dirt from a construction site each hour. how long will it take them to remove 36 tons of dirt from the site? (Answer needs to be in simplest form) how many solutions are there 14x + 6 = 2(5 + 7x) - 4 Is this rational or irrational I really need help ASAP! Will mark brainlist.